开发者:上海品职教育科技有限公司 隐私政策详情

应用版本:4.2.11(IOS)|3.2.5(安卓)APP下载

xujii · 2022年10月16日

通胀预期那项不加了吗?

* 问题详情,请 查看题干

NO.PZ202206070100000502

问题如下:

Using the data in Exhibit 3 and the investment team’s approach to predict the Fed’s next move, the new fed funds rate will most likely be:

选项:

A.2.9%. B.2.6%. C.2.1%.

解释:

Solution

C is correct. The Taylor rule is

Roptimal = Rneutral + [0.5 × (GDPgforecast – GDPgtrend)] + [0.5 × (IforecastItarget)]

= 2.5 + [0.5 × (3.0 – 4.5)] + [0.5 × (3.2 – 2.5)]

= 2.5 – 0.75 + 0.35

= 2.10%


C是正确的,依据泰勒公式可得:

Roptimal = Rneutral + [0.5 × (GDPgforecast – GDPgtrend)] + [0.5 × (IforecastItarget)]

= 2.5 + [0.5 × (3.0 – 4.5)] + [0.5 × (3.2 – 2.5)]

= 2.5 – 0.75 + 0.35

= 2.10%


通胀预期那项不加了吗?

1 个答案

源_品职助教 · 2022年10月17日

嗨,努力学习的PZer你好:


通胀项是需要加的。

这题是协会PRACTICE EXAM公布的题目。

题目应该还是按照教材修改前的思路给出的答案。

现在到了考场,是需要加上通胀这一项的。


----------------------------------------------
就算太阳没有迎着我们而来,我们正在朝着它而去,加油!

  • 1

    回答
  • 0

    关注
  • 375

    浏览
相关问题

NO.PZ202206070100000502 问题如下 Using the ta in Exhibit 1 anthe investment team’s approato prethe Fes next move, the new fefun rate will most likely be: A.2.9%. B.2.6%. C.2.1%. SolutionC is correct. The Taylor rule isRoptim= Rneutr+ [0.5 × (Ggforecast – Ggtren] + [0.5 × (Iforecast – Itarget)] = 2.5 + [0.5 × (3.0 – 4.5)] + [0.5 × (3.2 – 2.5)] = 2.5 – 0.75 + 0.35 = 2.10%C是正确的,依据泰勒公式可得Roptim= Rneutr+ [0.5 × (Ggforecast – Ggtren] + [0.5 × (Iforecast – Itarget)] = 2.5 + [0.5 × (3.0 – 4.5)] + [0.5 × (3.2 – 2.5)] = 2.5 – 0.75 + 0.35 = 2.10% 对于加不加expecteinflation这项先说下我的理解题目条件给的是FFR neutrrate, 问的是new FFR neutrrate。第一种情况假设题目给的FFR neutrrate是rerate, 那么问的也就是rerate,因为表述方式是一样的,那么结果不需要加expecteinflation第二种情况假设题目给的FFR neutrrate是nominrate, 那么已经包含了expecteinflation, 最后问我们的也是nominrate, 也不需要再重复加expecteinflation.综上,我认为无论是哪种情况,都不需要另外添加expecteinflation, 除非题目特别明确的给出rerate然后问nominrate所以,我认为这题答案并没有错误,老师认为呢?

2024-01-31 16:38 2 · 回答

NO.PZ202206070100000502问题如下 Using the ta in Exhibit 1 anthe investment team’s approato prethe Fes next move, the new fefun rate will most likely be: A.2.9%.B.2.6%.C.2.1%. SolutionC is correct. The Taylor rule isRoptim= Rneutr+ [0.5 × (Ggforecast – Ggtren] + [0.5 × (Iforecast – Itarget)] = 2.5 + [0.5 × (3.0 – 4.5)] + [0.5 × (3.2 – 2.5)] = 2.5 – 0.75 + 0.35 = 2.10%C是正确的,依据泰勒公式可得Roptim= Rneutr+ [0.5 × (Ggforecast – Ggtren] + [0.5 × (Iforecast – Itarget)] = 2.5 + [0.5 × (3.0 – 4.5)] + [0.5 × (3.2 – 2.5)] = 2.5 – 0.75 + 0.35 = 2.10% 按照目前的公式计算,最终结果应该是2.1%+3.2%=5.3%吗?如果题目中没有给出neutrrate是re还是nominal,是按照默认为rerate吗?

2024-01-04 16:33 1 · 回答

NO.PZ202206070100000502问题如下 Using the ta in Exhibit 1 anthe investment team’s approato prethe Fes next move, the new fefun rate will most likely be: A.2.9%.B.2.6%.C.2.1%. SolutionC is correct. The Taylor rule isRoptim= Rneutr+ [0.5 × (Ggforecast – Ggtren] + [0.5 × (Iforecast – Itarget)] = 2.5 + [0.5 × (3.0 – 4.5)] + [0.5 × (3.2 – 2.5)] = 2.5 – 0.75 + 0.35 = 2.10%C是正确的,依据泰勒公式可得Roptim= Rneutr+ [0.5 × (Ggforecast – Ggtren] + [0.5 × (Iforecast – Itarget)] = 2.5 + [0.5 × (3.0 – 4.5)] + [0.5 × (3.2 – 2.5)] = 2.5 – 0.75 + 0.35 = 2.10% 明知道答案错了,为什么不改下答案内容。这样容易误导

2023-08-03 09:07 1 · 回答

NO.PZ202206070100000502 问题如下 Using the ta in Exhibit 1 anthe investment team’s approato prethe Fes next move, the new fefun rate will most likely be: A.2.9%. B.2.6%. C.2.1%. SolutionC is correct. The Taylor rule isRoptim= Rneutr+ [0.5 × (Ggforecast – Ggtren] + [0.5 × (Iforecast – Itarget)] = 2.5 + [0.5 × (3.0 – 4.5)] + [0.5 × (3.2 – 2.5)] = 2.5 – 0.75 + 0.35 = 2.10%C是正确的,依据泰勒公式可得Roptim= Rneutr+ [0.5 × (Ggforecast – Ggtren] + [0.5 × (Iforecast – Itarget)] = 2.5 + [0.5 × (3.0 – 4.5)] + [0.5 × (3.2 – 2.5)] = 2.5 – 0.75 + 0.35 = 2.10% 老师请问,根据现在的考纲Taylor Rule需要加上ExpecteInflation, 那在这个题干里,Inflation Expecte= Inflation Forecast,是吗?

2022-12-20 16:07 1 · 回答

NO.PZ202206070100000502 问题如下 Using the ta in Exhibit 3 anthe investment team’s approato prethe Fes next move, the new fefun rate will most likely be: A.2.9%. B.2.6%. C.2.1%. SolutionC is correct. The Taylor rule isRoptim= Rneutr+ [0.5 × (Ggforecast – Ggtren] + [0.5 × (Iforecast – Itarget)] = 2.5 + [0.5 × (3.0 – 4.5)] + [0.5 × (3.2 – 2.5)] = 2.5 – 0.75 + 0.35 = 2.10%C是正确的,依据泰勒公式可得Roptim= Rneutr+ [0.5 × (Ggforecast – Ggtren] + [0.5 × (Iforecast – Itarget)] = 2.5 + [0.5 × (3.0 – 4.5)] + [0.5 × (3.2 – 2.5)] = 2.5 – 0.75 + 0.35 = 2.10% 想问一下,current rate和neutrrate的区别是,current=neutral+inflation吗?考试的时候需要加inflation,如果没有单独给出,是不是用current rate带入taylor公式呢?

2022-11-11 12:52 1 · 回答